Direct sum of closed Orthogonal Subspaces












2














Given a sequence ${mathscr{H}_n}_{n=1}^{infty}$ of closed, orthogonal subspaces of a Hilbert Space $mathscr{H}$, we define the infinite direct sum to be:
$$
bigoplus_{n = 1}^{infty} mathscr{H}_n = left {sum_{n = 1}^infty x_n : x_n in mathscr{H}_n, sum_{n = 1}^infty|x_n|^2 < inftyright }
$$

The question asks me to prove that this is a closed subspace of $mathscr{H}$.



The right hand side condition makes sense to me as for orthogonal $x_n$ we have $|sum x_n|^2 = sum |x_n|^2$. For a sum of two elements in the space, we see that:
begin{align*}
sum_{n = 1}^inftyleft |x_n+ y_n right |^2 &leq sum_{n = 1}^infty (|x_n| + |y_n|)^2 \ &= sum_{n = 1}^infty (|x_n|^2 + 2|x_ny_n| + |y_n|^2) \ &leq sum_{n = 1}^infty (|x_n|^2 + |y_n|^2)+ 2left(sum_{n = 1}^infty|x_n|^2right)^{1/2} left(sum_{n = 1}^{infty} |y_n|^2right)^{1/2} \
&<infty
end{align*}

Thus, a sum of two elements is also a member of the set. the set is also clearly closed under scalar multiplication. How would I prove it is closed? Exactly why can we take sequences and show they converge in the set?










share|cite|improve this question



























    2














    Given a sequence ${mathscr{H}_n}_{n=1}^{infty}$ of closed, orthogonal subspaces of a Hilbert Space $mathscr{H}$, we define the infinite direct sum to be:
    $$
    bigoplus_{n = 1}^{infty} mathscr{H}_n = left {sum_{n = 1}^infty x_n : x_n in mathscr{H}_n, sum_{n = 1}^infty|x_n|^2 < inftyright }
    $$

    The question asks me to prove that this is a closed subspace of $mathscr{H}$.



    The right hand side condition makes sense to me as for orthogonal $x_n$ we have $|sum x_n|^2 = sum |x_n|^2$. For a sum of two elements in the space, we see that:
    begin{align*}
    sum_{n = 1}^inftyleft |x_n+ y_n right |^2 &leq sum_{n = 1}^infty (|x_n| + |y_n|)^2 \ &= sum_{n = 1}^infty (|x_n|^2 + 2|x_ny_n| + |y_n|^2) \ &leq sum_{n = 1}^infty (|x_n|^2 + |y_n|^2)+ 2left(sum_{n = 1}^infty|x_n|^2right)^{1/2} left(sum_{n = 1}^{infty} |y_n|^2right)^{1/2} \
    &<infty
    end{align*}

    Thus, a sum of two elements is also a member of the set. the set is also clearly closed under scalar multiplication. How would I prove it is closed? Exactly why can we take sequences and show they converge in the set?










    share|cite|improve this question

























      2












      2








      2







      Given a sequence ${mathscr{H}_n}_{n=1}^{infty}$ of closed, orthogonal subspaces of a Hilbert Space $mathscr{H}$, we define the infinite direct sum to be:
      $$
      bigoplus_{n = 1}^{infty} mathscr{H}_n = left {sum_{n = 1}^infty x_n : x_n in mathscr{H}_n, sum_{n = 1}^infty|x_n|^2 < inftyright }
      $$

      The question asks me to prove that this is a closed subspace of $mathscr{H}$.



      The right hand side condition makes sense to me as for orthogonal $x_n$ we have $|sum x_n|^2 = sum |x_n|^2$. For a sum of two elements in the space, we see that:
      begin{align*}
      sum_{n = 1}^inftyleft |x_n+ y_n right |^2 &leq sum_{n = 1}^infty (|x_n| + |y_n|)^2 \ &= sum_{n = 1}^infty (|x_n|^2 + 2|x_ny_n| + |y_n|^2) \ &leq sum_{n = 1}^infty (|x_n|^2 + |y_n|^2)+ 2left(sum_{n = 1}^infty|x_n|^2right)^{1/2} left(sum_{n = 1}^{infty} |y_n|^2right)^{1/2} \
      &<infty
      end{align*}

      Thus, a sum of two elements is also a member of the set. the set is also clearly closed under scalar multiplication. How would I prove it is closed? Exactly why can we take sequences and show they converge in the set?










      share|cite|improve this question













      Given a sequence ${mathscr{H}_n}_{n=1}^{infty}$ of closed, orthogonal subspaces of a Hilbert Space $mathscr{H}$, we define the infinite direct sum to be:
      $$
      bigoplus_{n = 1}^{infty} mathscr{H}_n = left {sum_{n = 1}^infty x_n : x_n in mathscr{H}_n, sum_{n = 1}^infty|x_n|^2 < inftyright }
      $$

      The question asks me to prove that this is a closed subspace of $mathscr{H}$.



      The right hand side condition makes sense to me as for orthogonal $x_n$ we have $|sum x_n|^2 = sum |x_n|^2$. For a sum of two elements in the space, we see that:
      begin{align*}
      sum_{n = 1}^inftyleft |x_n+ y_n right |^2 &leq sum_{n = 1}^infty (|x_n| + |y_n|)^2 \ &= sum_{n = 1}^infty (|x_n|^2 + 2|x_ny_n| + |y_n|^2) \ &leq sum_{n = 1}^infty (|x_n|^2 + |y_n|^2)+ 2left(sum_{n = 1}^infty|x_n|^2right)^{1/2} left(sum_{n = 1}^{infty} |y_n|^2right)^{1/2} \
      &<infty
      end{align*}

      Thus, a sum of two elements is also a member of the set. the set is also clearly closed under scalar multiplication. How would I prove it is closed? Exactly why can we take sequences and show they converge in the set?







      functional-analysis hilbert-spaces






      share|cite|improve this question













      share|cite|improve this question











      share|cite|improve this question




      share|cite|improve this question










      asked Dec 1 at 4:46









      rubikscube09

      1,169717




      1,169717






















          2 Answers
          2






          active

          oldest

          votes


















          3














          Let $(y_n)_{ n in mathbb{N}}$ be a Cauchy-sequence in $bigoplus_{n = 1}^{infty} mathscr{H}_n$. Write $y_n = sum_{m=1}^infty x_{m,n}$ with $x_{m,n} in mathscr{H}_m$ and note that by using the orthogonality we obtain
          $$sum_{m=1}^infty |x_{n,m} - x_{n',m}|^2 = |y_n -y_{n'}|^2.$$
          Thus $(x_{n,m})_n$ is also a Cauchy-sequence and since $mathcal{H}_n$ is complete (as a closed set of a complete space), we get that $x_{n,m} rightarrow x_m in mathscr{H}_m$. Next, we show that the sum $sum_{m=1}^infty x_m$ is convergent. (Here we need to argue that we can interchange the limes and the infinite summation.) For this, note that $|y_n - y_1|$ is bounded, say by $M$ and thus
          $$sum_{m=1}^k |x_{m} - x_{1,m}|^2 = lim_{nrightarrow infty} sum_{m=1}^k |x_{n,m} - x_{1,m}|^2 le limsup_{n rightarrow infty} |y_n-y_1| le M^2.$$
          Hence the last series (on the left hand side) is convergent, because it is bounded. By the $Delta$-inequality we also conclude that $$sum_{m=1}^infty |x_m|^2 <infty.$$ Since $mathscr{H}$ is complete and $(x_m)_m$ are orthogonal, we get that $y = sum_{m=1}^infty x_m$ is convergent in $mathscr{H}$ and by definition we also have $y in bigoplus_{n = 1}^{infty} mathscr{H}_n$. We can take $N in mathbb{N}$ so large that $|y_n -y_{n'}| < varepsilon$ for all $n,n' ge N$. Thus
          $$sum_{m=1}^k |x_{m} - x_{n',m}|^2 = lim_{nrightarrow infty} sum_{m=1}^k |x_{n,m} - x_{n',m}|^2 le limsup_{n rightarrow infty} |y_n-y_{n'}|^2 le varepsilon^2$$
          for all $n' ge N$. Letting $k rightarrow infty$ shows that
          $$|y-y_{n'}|^2 = sum_{m=1}^infty |x_{m} - x_{n',m}|^2 varepsilon^2$$
          for all $n' ge N$. Hence $(y_n)_n$ is convegent in $bigoplus_{n = 1}^{infty} mathscr{H}_n$.






          share|cite|improve this answer























          • Thanks for the answer. I am not really understanding the part where you bounded the sum. What was the reason for doing so?,
            – rubikscube09
            Dec 1 at 19:06










          • Ah I see, you were showing that the sum of squares of the limit sequence $sum |x_k|^2$ was finite. But couldn't you just say that $y_n$ is cauchy and thus norm bounded?
            – rubikscube09
            Dec 1 at 21:04










          • I want to show that $sum_{k=1}^infty |x_m| < infty$ and that is archived by (first) considering a finite sum in order to use the properties of limes (linearity). I.e. we have to argue that we can interchange the limes and the summation. (You can also use Fatou's lemma here for the counting measure on $mathbb{N}$.) I have extended my answer!
            – p4sch
            Dec 1 at 21:23










          • I used the monotone convergence theorem. I think that works as well. Thanks.
            – rubikscube09
            Dec 1 at 21:23










          • Yes, indeed! You can also use the monotone convergence theorem.
            – p4sch
            Dec 1 at 21:29



















          1














          As in the answer by @p4sch, let $(y_n)_n$ be a Cauchy sequence in $bigoplus_{n = 1}^{infty} mathscr{H}_n$ with $y_n = sum_{m=1}^infty x_{m,n}$ and $x_{m,n} in mathscr{H}_m$.



          For every $m in mathbb{N}$ we have



          $$|x_{m,k} - x_{m,j}|^2 le sum_{m=1}^infty |x_{m,k} - x_{m,j}|^2 = |y_k - y_j|^2 xrightarrow{k,jtoinfty} 0$$
          so $(x_{m,k})_k$ is Cauchy in $mathscr{H}_m$. Since $mathscr{H}_m$ is complete, there exists $x_{m,0} in mathscr{H}_m$ such that $x_{m,k} xrightarrow{ktoinfty} x_m$.



          Let $varepsilon > 0$ and pick $N in mathbb{N}$ such that $$k,j ge N implies sum_{m=1}^infty |x_{m,k} - x_{m,j}|^2 = |y_k - y_j|^2 < fracvarepsilon2$$



          In particular, assuming $k,j ge N$ for any $K in mathbb{N}$ we have $$sum_{m=1}^K|x_{m,k} - x_{m,j}|^2 < fracvarepsilon2$$



          Letting $k to infty$ implies
          $$sum_{m=1}^K|x_{m,0} - x_{m,j}|^2 le fracvarepsilon2$$
          and since $K$ was arbitrary, it follows
          $$sum_{m=1}^infty|x_{m,0} - x_{m,j}|^2 le fracvarepsilon2tag{$*$}$$



          Now we have
          $$sum_{m=1}^infty |x_{m,0}|^2 le sum_{m=1}^infty (|x_{m,0}-x_{m,j}| + |x_{m,j}|)^2 le 2left(sum_{m=1}^infty |x_{m,0}-x_{m,j}|^2 + sum_{m=1}^infty|x_{m,j}|^2right) < +infty$$



          Hence for $r,s in mathbb{N}$ we have $$left|sum_{m=r}^s x_{m,0}right|^2 = sum_{m=r}^s|x_{m,0}|^2 le sum_{m=r}^infty |x_{m,0}|^2
          xrightarrow{r,s to infty} 0$$

          so by completeness $y_0 := sum_{m=1}^infty x_{m,0}$ converges in $mathscr{H}$ and by $(*)$ we have $y_0 in bigoplus_{n = 1}^{infty} mathscr{H}_n$.



          $(*)$ also means $$j ge N implies |y_0 - y_{m,j}| le fracvarepsilon2 < varepsilon$$ which means $y_{j} xrightarrow{jtoinfty} y_0$.






          share|cite|improve this answer























            Your Answer





            StackExchange.ifUsing("editor", function () {
            return StackExchange.using("mathjaxEditing", function () {
            StackExchange.MarkdownEditor.creationCallbacks.add(function (editor, postfix) {
            StackExchange.mathjaxEditing.prepareWmdForMathJax(editor, postfix, [["$", "$"], ["\\(","\\)"]]);
            });
            });
            }, "mathjax-editing");

            StackExchange.ready(function() {
            var channelOptions = {
            tags: "".split(" "),
            id: "69"
            };
            initTagRenderer("".split(" "), "".split(" "), channelOptions);

            StackExchange.using("externalEditor", function() {
            // Have to fire editor after snippets, if snippets enabled
            if (StackExchange.settings.snippets.snippetsEnabled) {
            StackExchange.using("snippets", function() {
            createEditor();
            });
            }
            else {
            createEditor();
            }
            });

            function createEditor() {
            StackExchange.prepareEditor({
            heartbeatType: 'answer',
            autoActivateHeartbeat: false,
            convertImagesToLinks: true,
            noModals: true,
            showLowRepImageUploadWarning: true,
            reputationToPostImages: 10,
            bindNavPrevention: true,
            postfix: "",
            imageUploader: {
            brandingHtml: "Powered by u003ca class="icon-imgur-white" href="https://imgur.com/"u003eu003c/au003e",
            contentPolicyHtml: "User contributions licensed under u003ca href="https://creativecommons.org/licenses/by-sa/3.0/"u003ecc by-sa 3.0 with attribution requiredu003c/au003e u003ca href="https://stackoverflow.com/legal/content-policy"u003e(content policy)u003c/au003e",
            allowUrls: true
            },
            noCode: true, onDemand: true,
            discardSelector: ".discard-answer"
            ,immediatelyShowMarkdownHelp:true
            });


            }
            });














            draft saved

            draft discarded


















            StackExchange.ready(
            function () {
            StackExchange.openid.initPostLogin('.new-post-login', 'https%3a%2f%2fmath.stackexchange.com%2fquestions%2f3020999%2fdirect-sum-of-closed-orthogonal-subspaces%23new-answer', 'question_page');
            }
            );

            Post as a guest















            Required, but never shown

























            2 Answers
            2






            active

            oldest

            votes








            2 Answers
            2






            active

            oldest

            votes









            active

            oldest

            votes






            active

            oldest

            votes









            3














            Let $(y_n)_{ n in mathbb{N}}$ be a Cauchy-sequence in $bigoplus_{n = 1}^{infty} mathscr{H}_n$. Write $y_n = sum_{m=1}^infty x_{m,n}$ with $x_{m,n} in mathscr{H}_m$ and note that by using the orthogonality we obtain
            $$sum_{m=1}^infty |x_{n,m} - x_{n',m}|^2 = |y_n -y_{n'}|^2.$$
            Thus $(x_{n,m})_n$ is also a Cauchy-sequence and since $mathcal{H}_n$ is complete (as a closed set of a complete space), we get that $x_{n,m} rightarrow x_m in mathscr{H}_m$. Next, we show that the sum $sum_{m=1}^infty x_m$ is convergent. (Here we need to argue that we can interchange the limes and the infinite summation.) For this, note that $|y_n - y_1|$ is bounded, say by $M$ and thus
            $$sum_{m=1}^k |x_{m} - x_{1,m}|^2 = lim_{nrightarrow infty} sum_{m=1}^k |x_{n,m} - x_{1,m}|^2 le limsup_{n rightarrow infty} |y_n-y_1| le M^2.$$
            Hence the last series (on the left hand side) is convergent, because it is bounded. By the $Delta$-inequality we also conclude that $$sum_{m=1}^infty |x_m|^2 <infty.$$ Since $mathscr{H}$ is complete and $(x_m)_m$ are orthogonal, we get that $y = sum_{m=1}^infty x_m$ is convergent in $mathscr{H}$ and by definition we also have $y in bigoplus_{n = 1}^{infty} mathscr{H}_n$. We can take $N in mathbb{N}$ so large that $|y_n -y_{n'}| < varepsilon$ for all $n,n' ge N$. Thus
            $$sum_{m=1}^k |x_{m} - x_{n',m}|^2 = lim_{nrightarrow infty} sum_{m=1}^k |x_{n,m} - x_{n',m}|^2 le limsup_{n rightarrow infty} |y_n-y_{n'}|^2 le varepsilon^2$$
            for all $n' ge N$. Letting $k rightarrow infty$ shows that
            $$|y-y_{n'}|^2 = sum_{m=1}^infty |x_{m} - x_{n',m}|^2 varepsilon^2$$
            for all $n' ge N$. Hence $(y_n)_n$ is convegent in $bigoplus_{n = 1}^{infty} mathscr{H}_n$.






            share|cite|improve this answer























            • Thanks for the answer. I am not really understanding the part where you bounded the sum. What was the reason for doing so?,
              – rubikscube09
              Dec 1 at 19:06










            • Ah I see, you were showing that the sum of squares of the limit sequence $sum |x_k|^2$ was finite. But couldn't you just say that $y_n$ is cauchy and thus norm bounded?
              – rubikscube09
              Dec 1 at 21:04










            • I want to show that $sum_{k=1}^infty |x_m| < infty$ and that is archived by (first) considering a finite sum in order to use the properties of limes (linearity). I.e. we have to argue that we can interchange the limes and the summation. (You can also use Fatou's lemma here for the counting measure on $mathbb{N}$.) I have extended my answer!
              – p4sch
              Dec 1 at 21:23










            • I used the monotone convergence theorem. I think that works as well. Thanks.
              – rubikscube09
              Dec 1 at 21:23










            • Yes, indeed! You can also use the monotone convergence theorem.
              – p4sch
              Dec 1 at 21:29
















            3














            Let $(y_n)_{ n in mathbb{N}}$ be a Cauchy-sequence in $bigoplus_{n = 1}^{infty} mathscr{H}_n$. Write $y_n = sum_{m=1}^infty x_{m,n}$ with $x_{m,n} in mathscr{H}_m$ and note that by using the orthogonality we obtain
            $$sum_{m=1}^infty |x_{n,m} - x_{n',m}|^2 = |y_n -y_{n'}|^2.$$
            Thus $(x_{n,m})_n$ is also a Cauchy-sequence and since $mathcal{H}_n$ is complete (as a closed set of a complete space), we get that $x_{n,m} rightarrow x_m in mathscr{H}_m$. Next, we show that the sum $sum_{m=1}^infty x_m$ is convergent. (Here we need to argue that we can interchange the limes and the infinite summation.) For this, note that $|y_n - y_1|$ is bounded, say by $M$ and thus
            $$sum_{m=1}^k |x_{m} - x_{1,m}|^2 = lim_{nrightarrow infty} sum_{m=1}^k |x_{n,m} - x_{1,m}|^2 le limsup_{n rightarrow infty} |y_n-y_1| le M^2.$$
            Hence the last series (on the left hand side) is convergent, because it is bounded. By the $Delta$-inequality we also conclude that $$sum_{m=1}^infty |x_m|^2 <infty.$$ Since $mathscr{H}$ is complete and $(x_m)_m$ are orthogonal, we get that $y = sum_{m=1}^infty x_m$ is convergent in $mathscr{H}$ and by definition we also have $y in bigoplus_{n = 1}^{infty} mathscr{H}_n$. We can take $N in mathbb{N}$ so large that $|y_n -y_{n'}| < varepsilon$ for all $n,n' ge N$. Thus
            $$sum_{m=1}^k |x_{m} - x_{n',m}|^2 = lim_{nrightarrow infty} sum_{m=1}^k |x_{n,m} - x_{n',m}|^2 le limsup_{n rightarrow infty} |y_n-y_{n'}|^2 le varepsilon^2$$
            for all $n' ge N$. Letting $k rightarrow infty$ shows that
            $$|y-y_{n'}|^2 = sum_{m=1}^infty |x_{m} - x_{n',m}|^2 varepsilon^2$$
            for all $n' ge N$. Hence $(y_n)_n$ is convegent in $bigoplus_{n = 1}^{infty} mathscr{H}_n$.






            share|cite|improve this answer























            • Thanks for the answer. I am not really understanding the part where you bounded the sum. What was the reason for doing so?,
              – rubikscube09
              Dec 1 at 19:06










            • Ah I see, you were showing that the sum of squares of the limit sequence $sum |x_k|^2$ was finite. But couldn't you just say that $y_n$ is cauchy and thus norm bounded?
              – rubikscube09
              Dec 1 at 21:04










            • I want to show that $sum_{k=1}^infty |x_m| < infty$ and that is archived by (first) considering a finite sum in order to use the properties of limes (linearity). I.e. we have to argue that we can interchange the limes and the summation. (You can also use Fatou's lemma here for the counting measure on $mathbb{N}$.) I have extended my answer!
              – p4sch
              Dec 1 at 21:23










            • I used the monotone convergence theorem. I think that works as well. Thanks.
              – rubikscube09
              Dec 1 at 21:23










            • Yes, indeed! You can also use the monotone convergence theorem.
              – p4sch
              Dec 1 at 21:29














            3












            3








            3






            Let $(y_n)_{ n in mathbb{N}}$ be a Cauchy-sequence in $bigoplus_{n = 1}^{infty} mathscr{H}_n$. Write $y_n = sum_{m=1}^infty x_{m,n}$ with $x_{m,n} in mathscr{H}_m$ and note that by using the orthogonality we obtain
            $$sum_{m=1}^infty |x_{n,m} - x_{n',m}|^2 = |y_n -y_{n'}|^2.$$
            Thus $(x_{n,m})_n$ is also a Cauchy-sequence and since $mathcal{H}_n$ is complete (as a closed set of a complete space), we get that $x_{n,m} rightarrow x_m in mathscr{H}_m$. Next, we show that the sum $sum_{m=1}^infty x_m$ is convergent. (Here we need to argue that we can interchange the limes and the infinite summation.) For this, note that $|y_n - y_1|$ is bounded, say by $M$ and thus
            $$sum_{m=1}^k |x_{m} - x_{1,m}|^2 = lim_{nrightarrow infty} sum_{m=1}^k |x_{n,m} - x_{1,m}|^2 le limsup_{n rightarrow infty} |y_n-y_1| le M^2.$$
            Hence the last series (on the left hand side) is convergent, because it is bounded. By the $Delta$-inequality we also conclude that $$sum_{m=1}^infty |x_m|^2 <infty.$$ Since $mathscr{H}$ is complete and $(x_m)_m$ are orthogonal, we get that $y = sum_{m=1}^infty x_m$ is convergent in $mathscr{H}$ and by definition we also have $y in bigoplus_{n = 1}^{infty} mathscr{H}_n$. We can take $N in mathbb{N}$ so large that $|y_n -y_{n'}| < varepsilon$ for all $n,n' ge N$. Thus
            $$sum_{m=1}^k |x_{m} - x_{n',m}|^2 = lim_{nrightarrow infty} sum_{m=1}^k |x_{n,m} - x_{n',m}|^2 le limsup_{n rightarrow infty} |y_n-y_{n'}|^2 le varepsilon^2$$
            for all $n' ge N$. Letting $k rightarrow infty$ shows that
            $$|y-y_{n'}|^2 = sum_{m=1}^infty |x_{m} - x_{n',m}|^2 varepsilon^2$$
            for all $n' ge N$. Hence $(y_n)_n$ is convegent in $bigoplus_{n = 1}^{infty} mathscr{H}_n$.






            share|cite|improve this answer














            Let $(y_n)_{ n in mathbb{N}}$ be a Cauchy-sequence in $bigoplus_{n = 1}^{infty} mathscr{H}_n$. Write $y_n = sum_{m=1}^infty x_{m,n}$ with $x_{m,n} in mathscr{H}_m$ and note that by using the orthogonality we obtain
            $$sum_{m=1}^infty |x_{n,m} - x_{n',m}|^2 = |y_n -y_{n'}|^2.$$
            Thus $(x_{n,m})_n$ is also a Cauchy-sequence and since $mathcal{H}_n$ is complete (as a closed set of a complete space), we get that $x_{n,m} rightarrow x_m in mathscr{H}_m$. Next, we show that the sum $sum_{m=1}^infty x_m$ is convergent. (Here we need to argue that we can interchange the limes and the infinite summation.) For this, note that $|y_n - y_1|$ is bounded, say by $M$ and thus
            $$sum_{m=1}^k |x_{m} - x_{1,m}|^2 = lim_{nrightarrow infty} sum_{m=1}^k |x_{n,m} - x_{1,m}|^2 le limsup_{n rightarrow infty} |y_n-y_1| le M^2.$$
            Hence the last series (on the left hand side) is convergent, because it is bounded. By the $Delta$-inequality we also conclude that $$sum_{m=1}^infty |x_m|^2 <infty.$$ Since $mathscr{H}$ is complete and $(x_m)_m$ are orthogonal, we get that $y = sum_{m=1}^infty x_m$ is convergent in $mathscr{H}$ and by definition we also have $y in bigoplus_{n = 1}^{infty} mathscr{H}_n$. We can take $N in mathbb{N}$ so large that $|y_n -y_{n'}| < varepsilon$ for all $n,n' ge N$. Thus
            $$sum_{m=1}^k |x_{m} - x_{n',m}|^2 = lim_{nrightarrow infty} sum_{m=1}^k |x_{n,m} - x_{n',m}|^2 le limsup_{n rightarrow infty} |y_n-y_{n'}|^2 le varepsilon^2$$
            for all $n' ge N$. Letting $k rightarrow infty$ shows that
            $$|y-y_{n'}|^2 = sum_{m=1}^infty |x_{m} - x_{n',m}|^2 varepsilon^2$$
            for all $n' ge N$. Hence $(y_n)_n$ is convegent in $bigoplus_{n = 1}^{infty} mathscr{H}_n$.







            share|cite|improve this answer














            share|cite|improve this answer



            share|cite|improve this answer








            edited Dec 1 at 21:52

























            answered Dec 1 at 9:34









            p4sch

            4,760217




            4,760217












            • Thanks for the answer. I am not really understanding the part where you bounded the sum. What was the reason for doing so?,
              – rubikscube09
              Dec 1 at 19:06










            • Ah I see, you were showing that the sum of squares of the limit sequence $sum |x_k|^2$ was finite. But couldn't you just say that $y_n$ is cauchy and thus norm bounded?
              – rubikscube09
              Dec 1 at 21:04










            • I want to show that $sum_{k=1}^infty |x_m| < infty$ and that is archived by (first) considering a finite sum in order to use the properties of limes (linearity). I.e. we have to argue that we can interchange the limes and the summation. (You can also use Fatou's lemma here for the counting measure on $mathbb{N}$.) I have extended my answer!
              – p4sch
              Dec 1 at 21:23










            • I used the monotone convergence theorem. I think that works as well. Thanks.
              – rubikscube09
              Dec 1 at 21:23










            • Yes, indeed! You can also use the monotone convergence theorem.
              – p4sch
              Dec 1 at 21:29


















            • Thanks for the answer. I am not really understanding the part where you bounded the sum. What was the reason for doing so?,
              – rubikscube09
              Dec 1 at 19:06










            • Ah I see, you were showing that the sum of squares of the limit sequence $sum |x_k|^2$ was finite. But couldn't you just say that $y_n$ is cauchy and thus norm bounded?
              – rubikscube09
              Dec 1 at 21:04










            • I want to show that $sum_{k=1}^infty |x_m| < infty$ and that is archived by (first) considering a finite sum in order to use the properties of limes (linearity). I.e. we have to argue that we can interchange the limes and the summation. (You can also use Fatou's lemma here for the counting measure on $mathbb{N}$.) I have extended my answer!
              – p4sch
              Dec 1 at 21:23










            • I used the monotone convergence theorem. I think that works as well. Thanks.
              – rubikscube09
              Dec 1 at 21:23










            • Yes, indeed! You can also use the monotone convergence theorem.
              – p4sch
              Dec 1 at 21:29
















            Thanks for the answer. I am not really understanding the part where you bounded the sum. What was the reason for doing so?,
            – rubikscube09
            Dec 1 at 19:06




            Thanks for the answer. I am not really understanding the part where you bounded the sum. What was the reason for doing so?,
            – rubikscube09
            Dec 1 at 19:06












            Ah I see, you were showing that the sum of squares of the limit sequence $sum |x_k|^2$ was finite. But couldn't you just say that $y_n$ is cauchy and thus norm bounded?
            – rubikscube09
            Dec 1 at 21:04




            Ah I see, you were showing that the sum of squares of the limit sequence $sum |x_k|^2$ was finite. But couldn't you just say that $y_n$ is cauchy and thus norm bounded?
            – rubikscube09
            Dec 1 at 21:04












            I want to show that $sum_{k=1}^infty |x_m| < infty$ and that is archived by (first) considering a finite sum in order to use the properties of limes (linearity). I.e. we have to argue that we can interchange the limes and the summation. (You can also use Fatou's lemma here for the counting measure on $mathbb{N}$.) I have extended my answer!
            – p4sch
            Dec 1 at 21:23




            I want to show that $sum_{k=1}^infty |x_m| < infty$ and that is archived by (first) considering a finite sum in order to use the properties of limes (linearity). I.e. we have to argue that we can interchange the limes and the summation. (You can also use Fatou's lemma here for the counting measure on $mathbb{N}$.) I have extended my answer!
            – p4sch
            Dec 1 at 21:23












            I used the monotone convergence theorem. I think that works as well. Thanks.
            – rubikscube09
            Dec 1 at 21:23




            I used the monotone convergence theorem. I think that works as well. Thanks.
            – rubikscube09
            Dec 1 at 21:23












            Yes, indeed! You can also use the monotone convergence theorem.
            – p4sch
            Dec 1 at 21:29




            Yes, indeed! You can also use the monotone convergence theorem.
            – p4sch
            Dec 1 at 21:29











            1














            As in the answer by @p4sch, let $(y_n)_n$ be a Cauchy sequence in $bigoplus_{n = 1}^{infty} mathscr{H}_n$ with $y_n = sum_{m=1}^infty x_{m,n}$ and $x_{m,n} in mathscr{H}_m$.



            For every $m in mathbb{N}$ we have



            $$|x_{m,k} - x_{m,j}|^2 le sum_{m=1}^infty |x_{m,k} - x_{m,j}|^2 = |y_k - y_j|^2 xrightarrow{k,jtoinfty} 0$$
            so $(x_{m,k})_k$ is Cauchy in $mathscr{H}_m$. Since $mathscr{H}_m$ is complete, there exists $x_{m,0} in mathscr{H}_m$ such that $x_{m,k} xrightarrow{ktoinfty} x_m$.



            Let $varepsilon > 0$ and pick $N in mathbb{N}$ such that $$k,j ge N implies sum_{m=1}^infty |x_{m,k} - x_{m,j}|^2 = |y_k - y_j|^2 < fracvarepsilon2$$



            In particular, assuming $k,j ge N$ for any $K in mathbb{N}$ we have $$sum_{m=1}^K|x_{m,k} - x_{m,j}|^2 < fracvarepsilon2$$



            Letting $k to infty$ implies
            $$sum_{m=1}^K|x_{m,0} - x_{m,j}|^2 le fracvarepsilon2$$
            and since $K$ was arbitrary, it follows
            $$sum_{m=1}^infty|x_{m,0} - x_{m,j}|^2 le fracvarepsilon2tag{$*$}$$



            Now we have
            $$sum_{m=1}^infty |x_{m,0}|^2 le sum_{m=1}^infty (|x_{m,0}-x_{m,j}| + |x_{m,j}|)^2 le 2left(sum_{m=1}^infty |x_{m,0}-x_{m,j}|^2 + sum_{m=1}^infty|x_{m,j}|^2right) < +infty$$



            Hence for $r,s in mathbb{N}$ we have $$left|sum_{m=r}^s x_{m,0}right|^2 = sum_{m=r}^s|x_{m,0}|^2 le sum_{m=r}^infty |x_{m,0}|^2
            xrightarrow{r,s to infty} 0$$

            so by completeness $y_0 := sum_{m=1}^infty x_{m,0}$ converges in $mathscr{H}$ and by $(*)$ we have $y_0 in bigoplus_{n = 1}^{infty} mathscr{H}_n$.



            $(*)$ also means $$j ge N implies |y_0 - y_{m,j}| le fracvarepsilon2 < varepsilon$$ which means $y_{j} xrightarrow{jtoinfty} y_0$.






            share|cite|improve this answer




























              1














              As in the answer by @p4sch, let $(y_n)_n$ be a Cauchy sequence in $bigoplus_{n = 1}^{infty} mathscr{H}_n$ with $y_n = sum_{m=1}^infty x_{m,n}$ and $x_{m,n} in mathscr{H}_m$.



              For every $m in mathbb{N}$ we have



              $$|x_{m,k} - x_{m,j}|^2 le sum_{m=1}^infty |x_{m,k} - x_{m,j}|^2 = |y_k - y_j|^2 xrightarrow{k,jtoinfty} 0$$
              so $(x_{m,k})_k$ is Cauchy in $mathscr{H}_m$. Since $mathscr{H}_m$ is complete, there exists $x_{m,0} in mathscr{H}_m$ such that $x_{m,k} xrightarrow{ktoinfty} x_m$.



              Let $varepsilon > 0$ and pick $N in mathbb{N}$ such that $$k,j ge N implies sum_{m=1}^infty |x_{m,k} - x_{m,j}|^2 = |y_k - y_j|^2 < fracvarepsilon2$$



              In particular, assuming $k,j ge N$ for any $K in mathbb{N}$ we have $$sum_{m=1}^K|x_{m,k} - x_{m,j}|^2 < fracvarepsilon2$$



              Letting $k to infty$ implies
              $$sum_{m=1}^K|x_{m,0} - x_{m,j}|^2 le fracvarepsilon2$$
              and since $K$ was arbitrary, it follows
              $$sum_{m=1}^infty|x_{m,0} - x_{m,j}|^2 le fracvarepsilon2tag{$*$}$$



              Now we have
              $$sum_{m=1}^infty |x_{m,0}|^2 le sum_{m=1}^infty (|x_{m,0}-x_{m,j}| + |x_{m,j}|)^2 le 2left(sum_{m=1}^infty |x_{m,0}-x_{m,j}|^2 + sum_{m=1}^infty|x_{m,j}|^2right) < +infty$$



              Hence for $r,s in mathbb{N}$ we have $$left|sum_{m=r}^s x_{m,0}right|^2 = sum_{m=r}^s|x_{m,0}|^2 le sum_{m=r}^infty |x_{m,0}|^2
              xrightarrow{r,s to infty} 0$$

              so by completeness $y_0 := sum_{m=1}^infty x_{m,0}$ converges in $mathscr{H}$ and by $(*)$ we have $y_0 in bigoplus_{n = 1}^{infty} mathscr{H}_n$.



              $(*)$ also means $$j ge N implies |y_0 - y_{m,j}| le fracvarepsilon2 < varepsilon$$ which means $y_{j} xrightarrow{jtoinfty} y_0$.






              share|cite|improve this answer


























                1












                1








                1






                As in the answer by @p4sch, let $(y_n)_n$ be a Cauchy sequence in $bigoplus_{n = 1}^{infty} mathscr{H}_n$ with $y_n = sum_{m=1}^infty x_{m,n}$ and $x_{m,n} in mathscr{H}_m$.



                For every $m in mathbb{N}$ we have



                $$|x_{m,k} - x_{m,j}|^2 le sum_{m=1}^infty |x_{m,k} - x_{m,j}|^2 = |y_k - y_j|^2 xrightarrow{k,jtoinfty} 0$$
                so $(x_{m,k})_k$ is Cauchy in $mathscr{H}_m$. Since $mathscr{H}_m$ is complete, there exists $x_{m,0} in mathscr{H}_m$ such that $x_{m,k} xrightarrow{ktoinfty} x_m$.



                Let $varepsilon > 0$ and pick $N in mathbb{N}$ such that $$k,j ge N implies sum_{m=1}^infty |x_{m,k} - x_{m,j}|^2 = |y_k - y_j|^2 < fracvarepsilon2$$



                In particular, assuming $k,j ge N$ for any $K in mathbb{N}$ we have $$sum_{m=1}^K|x_{m,k} - x_{m,j}|^2 < fracvarepsilon2$$



                Letting $k to infty$ implies
                $$sum_{m=1}^K|x_{m,0} - x_{m,j}|^2 le fracvarepsilon2$$
                and since $K$ was arbitrary, it follows
                $$sum_{m=1}^infty|x_{m,0} - x_{m,j}|^2 le fracvarepsilon2tag{$*$}$$



                Now we have
                $$sum_{m=1}^infty |x_{m,0}|^2 le sum_{m=1}^infty (|x_{m,0}-x_{m,j}| + |x_{m,j}|)^2 le 2left(sum_{m=1}^infty |x_{m,0}-x_{m,j}|^2 + sum_{m=1}^infty|x_{m,j}|^2right) < +infty$$



                Hence for $r,s in mathbb{N}$ we have $$left|sum_{m=r}^s x_{m,0}right|^2 = sum_{m=r}^s|x_{m,0}|^2 le sum_{m=r}^infty |x_{m,0}|^2
                xrightarrow{r,s to infty} 0$$

                so by completeness $y_0 := sum_{m=1}^infty x_{m,0}$ converges in $mathscr{H}$ and by $(*)$ we have $y_0 in bigoplus_{n = 1}^{infty} mathscr{H}_n$.



                $(*)$ also means $$j ge N implies |y_0 - y_{m,j}| le fracvarepsilon2 < varepsilon$$ which means $y_{j} xrightarrow{jtoinfty} y_0$.






                share|cite|improve this answer














                As in the answer by @p4sch, let $(y_n)_n$ be a Cauchy sequence in $bigoplus_{n = 1}^{infty} mathscr{H}_n$ with $y_n = sum_{m=1}^infty x_{m,n}$ and $x_{m,n} in mathscr{H}_m$.



                For every $m in mathbb{N}$ we have



                $$|x_{m,k} - x_{m,j}|^2 le sum_{m=1}^infty |x_{m,k} - x_{m,j}|^2 = |y_k - y_j|^2 xrightarrow{k,jtoinfty} 0$$
                so $(x_{m,k})_k$ is Cauchy in $mathscr{H}_m$. Since $mathscr{H}_m$ is complete, there exists $x_{m,0} in mathscr{H}_m$ such that $x_{m,k} xrightarrow{ktoinfty} x_m$.



                Let $varepsilon > 0$ and pick $N in mathbb{N}$ such that $$k,j ge N implies sum_{m=1}^infty |x_{m,k} - x_{m,j}|^2 = |y_k - y_j|^2 < fracvarepsilon2$$



                In particular, assuming $k,j ge N$ for any $K in mathbb{N}$ we have $$sum_{m=1}^K|x_{m,k} - x_{m,j}|^2 < fracvarepsilon2$$



                Letting $k to infty$ implies
                $$sum_{m=1}^K|x_{m,0} - x_{m,j}|^2 le fracvarepsilon2$$
                and since $K$ was arbitrary, it follows
                $$sum_{m=1}^infty|x_{m,0} - x_{m,j}|^2 le fracvarepsilon2tag{$*$}$$



                Now we have
                $$sum_{m=1}^infty |x_{m,0}|^2 le sum_{m=1}^infty (|x_{m,0}-x_{m,j}| + |x_{m,j}|)^2 le 2left(sum_{m=1}^infty |x_{m,0}-x_{m,j}|^2 + sum_{m=1}^infty|x_{m,j}|^2right) < +infty$$



                Hence for $r,s in mathbb{N}$ we have $$left|sum_{m=r}^s x_{m,0}right|^2 = sum_{m=r}^s|x_{m,0}|^2 le sum_{m=r}^infty |x_{m,0}|^2
                xrightarrow{r,s to infty} 0$$

                so by completeness $y_0 := sum_{m=1}^infty x_{m,0}$ converges in $mathscr{H}$ and by $(*)$ we have $y_0 in bigoplus_{n = 1}^{infty} mathscr{H}_n$.



                $(*)$ also means $$j ge N implies |y_0 - y_{m,j}| le fracvarepsilon2 < varepsilon$$ which means $y_{j} xrightarrow{jtoinfty} y_0$.







                share|cite|improve this answer














                share|cite|improve this answer



                share|cite|improve this answer








                edited Dec 1 at 23:41

























                answered Dec 1 at 21:58









                mechanodroid

                26.1k62245




                26.1k62245






























                    draft saved

                    draft discarded




















































                    Thanks for contributing an answer to Mathematics Stack Exchange!


                    • Please be sure to answer the question. Provide details and share your research!

                    But avoid



                    • Asking for help, clarification, or responding to other answers.

                    • Making statements based on opinion; back them up with references or personal experience.


                    Use MathJax to format equations. MathJax reference.


                    To learn more, see our tips on writing great answers.





                    Some of your past answers have not been well-received, and you're in danger of being blocked from answering.


                    Please pay close attention to the following guidance:


                    • Please be sure to answer the question. Provide details and share your research!

                    But avoid



                    • Asking for help, clarification, or responding to other answers.

                    • Making statements based on opinion; back them up with references or personal experience.


                    To learn more, see our tips on writing great answers.




                    draft saved


                    draft discarded














                    StackExchange.ready(
                    function () {
                    StackExchange.openid.initPostLogin('.new-post-login', 'https%3a%2f%2fmath.stackexchange.com%2fquestions%2f3020999%2fdirect-sum-of-closed-orthogonal-subspaces%23new-answer', 'question_page');
                    }
                    );

                    Post as a guest















                    Required, but never shown





















































                    Required, but never shown














                    Required, but never shown












                    Required, but never shown







                    Required, but never shown

































                    Required, but never shown














                    Required, but never shown












                    Required, but never shown







                    Required, but never shown







                    Popular posts from this blog

                    To store a contact into the json file from server.js file using a class in NodeJS

                    Redirect URL with Chrome Remote Debugging Android Devices

                    Dieringhausen